LSAT and Law School Admissions Forum

Get expert LSAT preparation and law school admissions advice from PowerScore Test Preparation.

User avatar
 Dave Killoran
PowerScore Staff
  • PowerScore Staff
  • Posts: 5853
  • Joined: Mar 25, 2011
|
#26375
Complete Question Explanation
(The complete setup for this game can be found here: lsat/viewtopic.php?t=6413)

The correct answer choice is (D)

As discussed during the analysis of the third rule, Q cannot be given on Thursday morning as that ultimately creates a violation of the first rule. Thus, answer choice (D) is correct.
 egarcia193
  • Posts: 41
  • Joined: Jun 25, 2017
|
#38594
Hi,

I don't understand why Q can not go on Thursday morning if Q goes on Thursday morning then R and S go on Wednesday and J and K goes on Friday which leaves Q and N for Thursday which doesn't violate any rules so I'm confused about how D is correct?
User avatar
 Dave Killoran
PowerScore Staff
  • PowerScore Staff
  • Posts: 5853
  • Joined: Mar 25, 2011
|
#38596
Hi E,

Go through the rules again with your placements, and focus on what happens with the third rule (which fills Friday with K and N, not J and N as you suggested was possible), and then watch what happens with J. You're going to run out of room for the variables to conform to the three rules.

Whenever you reverse engineer a problem like this, and you know the answer is (D), there's going to be a rule violation somewhere along the line. It happens here too, so try again with the variable placements.

Good luck!

Get the most out of your LSAT Prep Plus subscription.

Analyze and track your performance with our Testing and Analytics Package.